LSAT and Law School Admissions Forum

Get expert LSAT preparation and law school admissions advice from PowerScore Test Preparation.

 Administrator
PowerScore Staff
  • PowerScore Staff
  • Posts: 8917
  • Joined: Feb 02, 2011
|
#35366
Complete Question Explanation

Weaken—#%. The correct answer choice is (D)

The key to answering this question correctly is to simplify the language in the stimulus and focus
on the numerical evidence presented in support of the conclusion. The author observes that libraries
spend a lot of money on subscriptions to scholarly journals. These subscription rates have increased
over the years, but the cost of publishing the journals has remained unchanged. This evidence leads
the author to conclude that publishing a scholarly journal is much more profitable now than it was in
the past.

The author’s argument is weak, but it is not inherently flawed. Indeed, if journals charge libraries
more money but keep their publishing costs the same, it is entirely possible that their profits are
also higher (economic profit being the difference between the total revenue and total cost). This line
of reasoning assumes, however, that scholarly journals rely on university libraries for a significant
portion of their revenue stream. If most subscribers were individuals, however, then the increased
subscription rate that a library pays would add little to the journals’ profits. Profits may be higher,
but not that much higher. Likewise, if fewer libraries subscribe to scholarly journals today than they
did in the past, journals would derive greater income from fewer customers, potentially generating
lower total revenue. So even if publishing costs remained unchanged, the profit margin may not be as
high as the author suspects.

Note that almost all correct Weaken question answers leave the premises intact and focus instead
on the conclusion of the argument. Thus, in prephrasing an answer, look to undermine the central
assumptions upon which the argument depends. In this particular instance, our job is to show why
scholarly journals may not be as profitable as the author suggests, even if they charge libraries more
money for subscriptions.

Answer choice (A): How libraries are able to pay for the higher subscription rates has no bearing on
the conclusion of the argument, which is about the profitability of scholarly journals, not libraries.

Answer choice (B): As with answer choice (A), the conclusion of the argument pertains to the
profitability of scholarly journals, not libraries. How libraries allocate their budget is irrelevant to the
issue at stake.

Answer choice (C): This answer choice attempts to justify the increased subscription rates charged
by scholarly journals, instead of showing why their profitability may be exaggerated. This is a Shell
Game answer, weakening a different conclusion. The author never claimed that scholarly journals
charge too much for their subscriptions, or that their subscription rates have increased beyond reason.

Answer choice (D): This is the correct answer choice. As discussed above, the author assumes that
scholarly journals rely on university libraries for a significant portion of their revenue stream. But if
most subscribers are individuals who pay the same subscription rates as before, then the rate increase
for libraries would not generate that much additional revenue. Consequently, we have a good reason
to question whether publishing scholarly journals is much more profitable now than it was in the
past.

Answer choice (E): How often scholarly journals are published has no bearing on the question of
whether their profitability has increased.
 ltoulme
  • Posts: 25
  • Joined: Feb 05, 2014
|
#30344
I thought (D) was incorrect because even if most subscribes are individuals, couldn't the subscription for libraries have "increased dramatically" enough to give journals more profits?

I chose (C) because I thought it showed that even though they charge libraries high subscriptions, the publishing scholarly journal business isn't actually that profitable.

Thanks very much!
 Adam Tyson
PowerScore Staff
  • PowerScore Staff
  • Posts: 5153
  • Joined: Apr 14, 2011
|
#30393
Thanks for the question, Itoulme, as this gives me a chance to talk about things like relative relationships ("much more" vs just "more", for example) and about the difference between weakening an argument and disproving it.

The stimulus here tells us that the average price of the subscriptions for libraries has gone up a lot ("increased dramatically"), and concludes that those journals must be making a lot more money than they were ("much more profitable"). That's what we need to weaken - that they are MUCH more profitable (emphasis added here). How do we do that? We find something in an answer choice that lessens the profit. We know from the stimulus that publishing costs have stayed about the same, so that can't be it. Maybe postage/shipping costs went way up? Maybe the taxes on those subscriptions went up? Those were the ideas bouncing around in my head as I went into the answer choices. Look for something that suggests that profits may not have gone up all that much.

Answer choice C is an irrelevant answer in that it doesn't give us a reason for the profits to be less than the author suggested they were. Instead, it sort of gives us a reason why the publishers might have raised the price, a motivation. Does "why" have anything to do with it? Not a bit - all we care about is showing that the profits may not be all that much.

Answer D does that by putting it all into perspective. The libraries may be paying much more than before but they only make up a small percentage of the total list of subscribers, and most subscribers are not paying any more than they were. Sure, profits may have gone up, but in this scenario it seems a little less likely that they are making MUCH MORE profits (just SOME more).

Answer D does not destroy the argument. Depending on things like what percentage of total subscriptions goes to libraries (less than half, sure, but 49% or 1% could make a big difference), and how much of an increase is dramatic, and how much profit were they making in the first place, our analysis could change, but at least we have introduced a factor that should make us reconsider the conclusion and ask for more evidence. It could still be that the publishers are making much more profits thanks to the increased rates for libraries. That's okay, though, because our goal in the question stem was not to destroy or disprove the argument, but merely to weaken it, to make it less likely. Sometimes a weaken answer will inflict major damage on an argument, but often, as here, it just introduces an element of doubt, a chink in the armor. That's enough! When the other answers do nothing at all, an answer that weakens a little is the best answer we have, and so we pick it and get credit for it. Isn't that great?

Take another look and see if that makes sense. If not, come back and ask for more help. We'll be here!
 ltoulme
  • Posts: 25
  • Joined: Feb 05, 2014
|
#30397
This makes perfect sense now - thanks very much!
User avatar
 PresidentLSAT
  • Posts: 87
  • Joined: Apr 19, 2021
|
#101970
Hi,

Hopefully I get an explanation on this. I would have chosen D if it said "libraries make up a small portion of subscribers of publishing house." D gives room to argue that the 49% possibility can make a significant difference in profit accumulation. What am I missing here?
 Rachael Wilkenfeld
PowerScore Staff
  • PowerScore Staff
  • Posts: 1358
  • Joined: Dec 15, 2011
|
#102003
Hi President,

Remember your goal on Weaken questions is not to completely destroy an argument; it's just to make it less likely, to drive a bit of a wedge between the premise(s) and the conclusion.

Here, even if we say that only 51% of subscribers are individuals, that would still make it less likely that the journals are raking in the profits. It won't disprove the conclusion. But it makes it less likely. The higher that 51% goes, the more likely the conclusion is to be questionable. The answer choice is unclear as to how much it weakens, but it definitely weakens the conclusion. Additionally, it's the only one of the five choices to draw any doubt on the conclusion.

Hope that helps.

Get the most out of your LSAT Prep Plus subscription.

Analyze and track your performance with our Testing and Analytics Package.